Verbal problems from the *free* official practice tests and
problems from mba.com
ashutosh22
Forum Guests
 
Posts: 3
Joined: Sat Nov 05, 2011 2:24 am
 

Re: CR: Technological improvements and reduced equipment costs

by ashutosh22 Sat Dec 03, 2011 5:42 am

Got it.

I was confused with delta changes.

Thanks Ron.
RonPurewal
Students
 
Posts: 19744
Joined: Tue Aug 14, 2007 8:23 am
 

Re: CR: Technological improvements and reduced equipment costs

by RonPurewal Sat Dec 10, 2011 5:04 am

sure.
gmatwork
Course Students
 
Posts: 185
Joined: Wed Dec 31, 1969 8:00 pm
 

Re: CR: Technological improvements and reduced equipment costs

by gmatwork Tue Oct 30, 2012 8:29 am

even after reading all the posts I still don't understand why the price or cost of oil wouldn't matter? Why is A wrong? I did read everything but am still confused. Plus, can you please explain the definition of economic viability.
tim
Course Students
 
Posts: 5665
Joined: Tue Sep 11, 2007 9:08 am
Location: Southwest Airlines, seat 21C
 

Re: CR: Technological improvements and reduced equipment costs

by tim Fri Nov 09, 2012 6:41 pm

first, if you feel you need an external definition of economic viability you are doing something wrong. these questions never require outside knowledge and instead ask you to use what's on the page, together with your thinking skills, to process the information. you'll notice in fact that the problem itself defines the threshold of economic viability! the reason A is wrong is because the threshold of economic viability is roughly defined as the price oil would NEED to be. the actual cost of oil will have no effect on the price oil would NEED to be. of course, veronica said as much over two years ago, so i'm afraid if this logic doesn't work for you and you are unwilling to accept it as true, there isn't much more we'll be able to do for you here..
Tim Sanders
Manhattan GMAT Instructor

Follow this link for some important tips to get the most out of your forum experience:
https://www.manhattanprep.com/gmat/forums/a-few-tips-t31405.html
saadfarooq
Course Students
 
Posts: 2
Joined: Sat May 05, 2012 1:26 am
 

Re: CR: Technological improvements and reduced equipment costs

by saadfarooq Fri Nov 23, 2012 6:00 am

I think the confusion stems from misinterpreting the question stem.

First time I read the question I thought the question was talking about the INCREASE in Oil price, however if you pay attention to the preposition in the phrase you realize that its not talking about "the price BY which oil would have to rise" but "the price TO which oil would have to rise"

"that is, the price per barrel TO which oil would have to rise"
tim
Course Students
 
Posts: 5665
Joined: Tue Sep 11, 2007 9:08 am
Location: Southwest Airlines, seat 21C
 

Re: CR: Technological improvements and reduced equipment costs

by tim Fri Nov 23, 2012 8:14 pm

notice how important those little words are! they can change the whole meaning of the problem..
Tim Sanders
Manhattan GMAT Instructor

Follow this link for some important tips to get the most out of your forum experience:
https://www.manhattanprep.com/gmat/forums/a-few-tips-t31405.html
divineacclivity
Forum Guests
 
Posts: 288
Joined: Sun Apr 01, 2012 4:09 am
 

Re: CR: Technological improvements and reduced equipment costs

by divineacclivity Sat Nov 24, 2012 3:36 am

My thought & understanding to explain the paradox:

In generating electricity in olden times (before technological advancements):
oil-fired plant efficiency [at oil price = $35] = solar plant efficiency
i.e. E(o=35) = E(s)

Modern times (after technological improvements in solar efficiency):
oil plant efficiency (at oil price = $35) = solar plant efficiency
i.e. still, E(o=35) = E(s) when E(s) is increased bcos of tech advancements
=> E(o) must've increased too

So, to maintain the equation, if right hand side increases, left hand side should also increase, keeping oil price @ $35

So, increase in oil-fired plant efficiency has a direct co-relation with the paradox than drop in the price of oil alone, unless the arguments explicitly tells us that decrease in oil price alone does lead to increase in oil plant efficiency. What I mean to say is, oil price drop can NOT guarantee efficiency rise though it is one of the factors affecting efficiency e.g. an oil price drop + a major saving through equipment/land cost could still increase the efficiency. So, oil price rise alone can't be the deciding factor unless stated otherwise)

Experts, please correct me if I'm wrong. Thank you very much.
RonPurewal
Students
 
Posts: 19744
Joined: Tue Aug 14, 2007 8:23 am
 

Re: CR: Technological improvements and reduced equipment costs

by RonPurewal Fri Nov 30, 2012 12:14 pm

divineacclivity Wrote:My thought & understanding to explain the paradox:

In generating electricity in olden times (before technological advancements):
oil-fired plant efficiency [at oil price = $35] = solar plant efficiency
i.e. E(o=35) = E(s)

Modern times (after technological improvements in solar efficiency):
oil plant efficiency (at oil price = $35) = solar plant efficiency
i.e. still, E(o=35) = E(s) when E(s) is increased bcos of tech advancements
=> E(o) must've increased too

So, to maintain the equation, if right hand side increases, left hand side should also increase, keeping oil price @ $35

So, increase in oil-fired plant efficiency has a direct co-relation with the paradox than drop in the price of oil alone, unless the arguments explicitly tells us that decrease in oil price alone does lead to increase in oil plant efficiency. What I mean to say is, oil price drop can NOT guarantee efficiency rise though it is one of the factors affecting efficiency e.g. an oil price drop + a major saving through equipment/land cost could still increase the efficiency. So, oil price rise alone can't be the deciding factor unless stated otherwise)

Experts, please correct me if I'm wrong. Thank you very much.


well, to be honest, i'm not really able to follow the math notation you've got up there.
but, the idea can basically be summarized as follows: there are basically two factors that affect the competitiveness of oil as measured against other sources of energy.
1/ the price of oil;
2/ the efficiency of oil-driven processes.
this particular argument deals with "the price ... to which oil would have to rise", so that takes factor #1 out of consideration. that leaves only factor #2.

if your math equations are meant to convey basically the same idea, then, yes.
jmuduke08
Course Students
 
Posts: 32
Joined: Wed Feb 15, 2012 6:04 am
 

Re: CR: Technological improvements and reduced equipment costs

by jmuduke08 Wed Mar 20, 2013 5:57 pm

Ron,

Why is E not a viable answer choice? Am I wrong to think that an increase in the supply of oil could be a reason why the price of oil has remained unchanged?
tim
Course Students
 
Posts: 5665
Joined: Tue Sep 11, 2007 9:08 am
Location: Southwest Airlines, seat 21C
 

Re: CR: Technological improvements and reduced equipment costs

by tim Thu Mar 21, 2013 12:12 am

This problem has nothing to do with the price of oil or the supply of oil, so we won't need to address the issue of whether an increase in supply would leave the price unchanged. That's not the real issue here. The approach I would suggest is go back to the question itself and pay attention to what is really important, namely the threshold of economic viability..
Tim Sanders
Manhattan GMAT Instructor

Follow this link for some important tips to get the most out of your forum experience:
https://www.manhattanprep.com/gmat/forums/a-few-tips-t31405.html
tim
Course Students
 
Posts: 5665
Joined: Tue Sep 11, 2007 9:08 am
Location: Southwest Airlines, seat 21C
 

Re: CR: Technological improvements and reduced equipment costs

by tim Sun May 05, 2013 12:52 am

This seems to be a popular (and quite misunderstood) problem, so I'm going to take one more stab at explaining this one:

The question is essentially asking why the threshold of economic viability has remained unchanged. In other words, why would the price of oil still have to hit $35 in order for solar powered plants to be a good idea?

A lot of people are asking why A isn't the correct answer. The best answer I can give is that the price of oil has nothing to do with how much the price *would* have to be in order for solar power to be viable. Consider this: if I have $20000 available to buy a car, the "threshold of economic viability" for me to buy a car is $20000. What if the price of a car is $30000? Doesn't matter, I still can't buy a car unless it's less than $20000. What if the price suddenly drops to $10000? Doesn't matter; even though I can now buy the car (yay), this price drop didn't change the fact that I could only buy a car if it were less than $20000. What would change my threshold of viability? NOTHING about the price of the car can change my threshold; my own personal threshold of $20000 would only be changed by something that had to do with my own purchasing power - I make more money, the bank gives me a better interest rate, etc.

In a similar sense, the reason the threshold of viability has not changed in this problem has nothing to do with the price of oil, but instead has to do with how cost-effective solar power is relative to oil power. C does this by providing a scenario in which efficiency increases in solar power are balanced by efficiency increases in oil power.
Tim Sanders
Manhattan GMAT Instructor

Follow this link for some important tips to get the most out of your forum experience:
https://www.manhattanprep.com/gmat/forums/a-few-tips-t31405.html
yuanhongzhi0830
Forum Guests
 
Posts: 13
Joined: Sat Aug 03, 2013 6:41 am
 

Re: CR: Technological improvements and reduced equipment costs

by yuanhongzhi0830 Thu Aug 22, 2013 5:51 am

tim Wrote:A lot of people are asking why A isn't the correct answer. The best answer I can give is that the price of oil has nothing to do with how much the price *would* have to be in order for solar power to be viable.

Sorry for bumping this again, but I am confused about the reasoning.
After reading this question again, I want to confirm a sentence in the question: "that is, the price per barrel to which oil would have to rise"---does it refer to an incremental amount ( like"increase by") or an absolute amount ??(like"increase to")
I think this is the mother of confusion for all of us here
mathewtoms.all
Forum Guests
 
Posts: 1
Joined: Sun Jun 16, 2013 8:12 pm
 

Re: CR: Technological improvements and reduced equipment costs

by mathewtoms.all Fri Aug 30, 2013 3:58 am

I just realised a very minute detail. The catch is in that the that threshold is the price per barrel to which oil would have to decrease and not the price per per barrel by which ...

This implies that whatever the cost of oil, it doesn't matter. The price of petrol would have to increase to increase to a certain amount, which hasn't happened. Hence option A does nothing to explain the paradox.

Ron gave a an analogy with regard to waist size of pants. I think that's the best way to look at this problem.
RonPurewal
Students
 
Posts: 19744
Joined: Tue Aug 14, 2007 8:23 am
 

Re: CR: Technological improvements and reduced equipment costs

by RonPurewal Mon Sep 09, 2013 10:33 am

mathewtoms.all Wrote:I just realised a very minute detail. The catch is in that the that threshold is the price per barrel to which oil would have to decrease and not the price per per barrel by which ...

This implies that whatever the cost of oil, it doesn't matter. The price of petrol would have to increase to increase to a certain amount, which hasn't happened. Hence option A does nothing to explain the paradox.

Ron gave a an analogy with regard to waist size of pants. I think that's the best way to look at this problem.


Analogies are always the best way to understand CR problems that otherwise confuse you.

By the way, you can also get insight into the situation here by noting that the quantity in question is called a "threshold". If you know what "threshold" means -- in this kind of context, a specific fixed value that must be crossed for something to happen -- then it should be clear that "by which" is the wrong interpretation, because that wouldn't be a threshold.
cherryturanic
Forum Guests
 
Posts: 2
Joined: Wed Sep 25, 2013 6:40 pm
 

Re: CR: Technological improvements and reduced equipment costs

by cherryturanic Thu Sep 26, 2013 2:09 am

RonPurewal Wrote:
lambandme Wrote:But I do not think a CR problem needs so much mathematical reasoning, I think I'm taking the long way here. So I wonder if you have any other easier ways to tackle this problem?


you don't need the exact mathematical reasoning on problems like this; you just have to understand the contributions of different factors.

in this case, the problem is saying that solar power has gotten more efficient, but also that there has been no change in the target price of oil to achieve price equity with solar power. the only way this can be true is if something else related to oil-driven power, aside from the actual price of oil, has become cheaper in line with the improvements in solar power.


Hi Ron,
I totally understand why the oil price is irrelevant. Because no matter the actual price it is, the oil price must be higher than $35 to make the solar more efficient.

But when I interpreted the threshold viability into my own word, I got the idea like this: the cost of solar is equal to the cost of oil when the oil price is $35. The formula is Cost S= Cost O (Oil price=35). And Cost O actually equal to Oil price*Oil consumption. Choice C says: the efficiency of the oil-power factory increased. That means the plants consume less oil. So the cost goes down, and the threshold doesn’t change. Choice A says: the oil price decrease. To me, if the oil price decrease, then the cost should decrease also. Thus the threshold keep unchanged. I thought both A and C are correct.

Where did I get wrong here?

Thank you.